- PowerScore Staff
- Posts: 5972
- Joined: Mar 25, 2011
- Thu Apr 07, 2022 1:59 pm
#94664
Complete Question Explanation
(The complete setup for this game can be found here: lsat/viewtopic.php?f=280&t=8537)
The correct answer choice is (C)
The question stem establishes that three French novels are selected:
Thus, there must be at least two remaining works in order to meet the minimum of five works. This fact eliminates answer choice (A), which would create a group of only four works.
Applying the first rule, the two remaining works can include at most one French work. This fact eliminates answer choices (B) and (D).
Because three novels have already been selected, from the second rule at most one additional novel can be selected. This eliminates answer choice (E).
Consequently, answer choice (C) is correct.
(The complete setup for this game can be found here: lsat/viewtopic.php?f=280&t=8537)
The correct answer choice is (C)
The question stem establishes that three French novels are selected:
Thus, there must be at least two remaining works in order to meet the minimum of five works. This fact eliminates answer choice (A), which would create a group of only four works.
Applying the first rule, the two remaining works can include at most one French work. This fact eliminates answer choices (B) and (D).
Because three novels have already been selected, from the second rule at most one additional novel can be selected. This eliminates answer choice (E).
Consequently, answer choice (C) is correct.
You do not have the required permissions to view the files attached to this post.
Dave Killoran
PowerScore Test Preparation
Follow me on X/Twitter at http://twitter.com/DaveKilloran
My LSAT Articles: http://blog.powerscore.com/lsat/author/dave-killoran
PowerScore Podcast: http://www.powerscore.com/lsat/podcast/
PowerScore Test Preparation
Follow me on X/Twitter at http://twitter.com/DaveKilloran
My LSAT Articles: http://blog.powerscore.com/lsat/author/dave-killoran
PowerScore Podcast: http://www.powerscore.com/lsat/podcast/